LSAT and Law School Admissions Forum

Get expert LSAT preparation and law school admissions advice from PowerScore Test Preparation.

 Administrator
PowerScore Staff
  • PowerScore Staff
  • Posts: 8916
  • Joined: Feb 02, 2011
|
#41621
Complete Question Explanation
(The complete setup for this game can be found here: lsat/viewtopic.php?t=15887)

The correct answer choice is (E)

This Justify question asks us to identify a condition that, if true, would fully determine the order in which the students perform. Questions of this type can be particularly challenging if you have not previously examined the variables (and rules) that are especially likely to restrict the solution of the game.

Let’s examine our initial diagram for points of restriction. Notice, for instance, the two dual options we were able to infer: either G or K must be first, and either H or F must be fourth. To fully determine the order in which the students perform, the correct answer choice must “settle” these dual options. In other words, it must determine—at the very least—whether K or G performs first, and also whether H or F performs fourth.

Answer choice (A): This answer choice is incorrect. Knowing the precise order of the variables in the HF rotating block is not enough to fully determine the order of all five variables, if only because we still do not know if K or G performs first, nor can we determine the placement of J.

Answer choice (B): This answer choice is attractive, because a GF block necessitates the inclusion of H immediately after F, thanks to the third rule. And, since G cannot perform first in this instance, K must be first. This solution seems promising, especially if you consider the local diagram for Question #2. In that diagram, a GFH block can fully determine the order of the five performances:
PT65_D11 LG Explanations_game_#1_#4_diagram 1.png
Recall, however, that the diagram for Question #2 was predicated on the condition that J performs earlier than G. However, what if J performs later than G? A GFH block would still be possible, but the solution would be different:
PT65_D11 LG Explanations_game_#1_#4_diagram 2.png
Clearly, then, a GF block is not sufficient to fully determine the order of all five of the performances, rendering answer choice (B) incorrect.

Answer choice (C): This answer choice is also attractive, because the last rule requires the inclusion of F immediately before H in the HJ block, creating a powerful FHJ block. However, the relative order of K and G is still unknown, which renders answer choice (C) incorrect.

Answer choice (D): This answer choice is similar to answer choice (C). The last rule requires the inclusion of F immediately after H in the JH block, creating a JHF block. Nevertheless, the relative order of K and G is still unknown, which renders answer choice (D) also incorrect.

Answer choice (E): This is the correct answer choice. If K performs immediately before F, the last rule creates a KFH block. Thanks to the first rule, G must perform before the KFH block. Additionally, since K must perform earlier than J (second rule), it follows that J must perform last:
PT65_D11 LG Explanations_game_#1_#4_diagram 3.png
Since answer choice (E) is sufficient to fully determine the order in which the students perform, it is the correct answer choice.

Access to the two templates outlined earlier would have made our decision even easier. Since only Template 1 allows for a KF block, the solution is already restricted to one of the two templates:
PT65_D11 LG Explanations_game_#1_#4_diagram 4.png
It should be clear from this Template that a KF block is only possible if K is second, and F—third. This resolves all dual options in that setup, proving that the game has only one possible solution.
You do not have the required permissions to view the files attached to this post.
 cardinal2017
  • Posts: 19
  • Joined: Oct 23, 2016
|
#34071
I solved this question by chance as putting in answer choices from (E) and luckily got it correct.

But I wonder if this type of 'fully determined if ~' question can be solved by more methodical approach.

Hope somebody can help with this type of question!

Thanks!
 Adam Tyson
PowerScore Staff
  • PowerScore Staff
  • Posts: 5153
  • Joined: Apr 14, 2011
|
#34081
Good question, Cardinal, and the answer is a very lawyerly one - "it depends." In some games you will have set up templates (broad sketches of what the game could look like that vary slightly based on a few highly restrictive rules) as part of your diagram, and a reference to those templates could help you determine which variables are powerful enough that just placing them in one space solves everything. This game doesn't appeal to me as a template game because there aren't any highly restricting rules of the sort we would typically like to see for that approach, and in fact the setup, while small with only five spaces and five variables, is still very flexible.

Another approach might be to refer to solutions created as you answered earlier questions. For example, for question 2 in this game you might have sketched out something like this:

K :longline: J :longline: G :longline: HF/FH

This setup shows that when J is before G, there are two possible solutions. That means that J before G will NOT fully determine the outcome, nor will K going first or J going second or G going third. Sadly, none of those were answer choices for question 4, so this prior mini-diagram wouldn't have been any help here.

You could skip a question like this and save it for last, hoping that later questions will yield helpful results. Again, though, not in this case.

What's left? Exactly what you did! I wouldn't call it a guess, though - I would call it trial and error, or working it out. Some of us say this is the kind of question you just have to "bulldoze through" or that you must "brute force" the solution. I would take a quick scan through the answer choices to see if any of them looked particularly powerful to me. As I do that, I would be thinking about fixing the order of the HF/FH block, since that is an unknown, and also thinking about K and G, the only two variables that could possibly be first in the order, because I need to lock down which one it will be here. Any answer that combines one of H and F with one of K and G will therefore really jump out at me. Answer E is the only one to make that pairing, so I would be likely to test it before I consider any of the other answers. Lo and behold, it works, and I am done with the question! That's not a guess, but rather a strategic approach to a question that is designed to waste a lot of time sketching out possible solutions.

Take a more strategic approach to analyzing answer choices so that if you have to just bulldoze through them, you are at least starting with the most attractive or interesting candidate, rather than just working from A to E, and you will find that these questions go a little faster and get a little easier. Use what you have learned from earlier questions and your global diagram and see where that gets you, too. Bring all of those tools to bear on the problem, and you'll be ready for anything.

Keep pounding!
 cfu1
  • Posts: 15
  • Joined: Mar 28, 2017
|
#34909
Adam, answer choice B involves G and F, which meets your "any answer that combines one of H and F with one of K and G will therefore really jump out at me" requirement. With both answer choices, wouldn't you be able to determine the order of performances?

For answer choice B, the order would be K G F H J
For answer choices E, the order would be G K F H J

I believe both options satisfy all the rules.

Thanks for your help!
 mbell33
  • Posts: 2
  • Joined: May 17, 2017
|
#35005
I agree with cfu1, both B and E seem to provide valid solutions.

In general, I used the logic that Adam described above when answering this question. Unfortunately for me, when scanning the answer choices I noticed answer B first. Since it provided a solution, I selected it and moved on. I cannot see any reason why choice B is incorrect. Any help is appreciated!
 mbell33
  • Posts: 2
  • Joined: May 17, 2017
|
#35058
After looking at the problem with a fresh set of eyes, I now see that answer B has two solutions: KGFHJ and KJGFH. Thus, it's out as an answer and only E is left. You need to fix K with either F or H, which forces G first and J last.

Of course, the KJGFH solution was used in question 2 and used in Adam's response above. A bit embarrassing to have missed it.
 AthenaDalton
PowerScore Staff
  • PowerScore Staff
  • Posts: 296
  • Joined: May 02, 2017
|
#35243
mbell,

You're correct -- good catch! Kudos on circling back to this tough game and re-thinking your answer; that's exactly the sort of approach you should continue to use in your studies.

--Athena Dalton

Get the most out of your LSAT Prep Plus subscription.

Analyze and track your performance with our Testing and Analytics Package.